Last visit was: 26 Apr 2024, 02:07 It is currently 26 Apr 2024, 02:07

Close
GMAT Club Daily Prep
Thank you for using the timer - this advanced tool can estimate your performance and suggest more practice questions. We have subscribed you to Daily Prep Questions via email.

Customized
for You

we will pick new questions that match your level based on your Timer History

Track
Your Progress

every week, we’ll send you an estimated GMAT score based on your performance

Practice
Pays

we will pick new questions that match your level based on your Timer History
Not interested in getting valuable practice questions and articles delivered to your email? No problem, unsubscribe here.
Close
Request Expert Reply
Confirm Cancel
SORT BY:
Date
Math Expert
Joined: 02 Sep 2009
Posts: 92929
Own Kudos [?]: 619095 [12]
Given Kudos: 81609
Send PM
Math Expert
Joined: 02 Sep 2009
Posts: 92929
Own Kudos [?]: 619095 [0]
Given Kudos: 81609
Send PM
avatar
Intern
Intern
Joined: 28 Oct 2014
Posts: 10
Own Kudos [?]: 4 [2]
Given Kudos: 5
GMAT 1: 710 Q47 V40
Send PM
avatar
Intern
Intern
Joined: 05 Nov 2015
Posts: 32
Own Kudos [?]: 46 [0]
Given Kudos: 21
Send PM
Re: V04-39 [#permalink]
I don't agree that the conclusion of the argument is "many people do not achieve their fitness goals because they burn fewer calories than they intend to burn". I believe that BF1 is the main position of argument. The final sentence "when people record their exercises, their actual calories burned more closely matches their reported calories burned." confirms that i feel.
Current Student
Joined: 18 Jun 2016
Posts: 221
Own Kudos [?]: 613 [2]
Given Kudos: 111
Location: United States (NY)
GMAT 1: 720 Q50 V38
GMAT 2: 750 Q49 V42
GPA: 4
WE:General Management (Other)
Send PM
Re: V04-39 [#permalink]
2
Kudos
RahulSingh13 wrote:
I don't agree that the conclusion of the argument is "many people do not achieve their fitness goals because they burn fewer calories than they intend to burn". I believe that BF1 is the main position of argument. The final sentence "when people record their exercises, their actual calories burned more closely matches their reported calories burned." confirms that i feel.

The conclusion that we are talking about is that of the Experts and since BF1 is result of a survey, it is not the conclusion of Experts.


Somebody please correct me if I am wrong but I think we can Cross Off B because BF1 is not the explanation of the Theory. As a matter of fact, it CANNOT be the Explanation as the Theory presented by experts is based on that result. Thinking about chronology,

Result of Study -> Theory by Experts -> Explanation/Another study.

We explain After we make a claim and not before it. So, Results of Study cannot be the explanation of Theory.
Manager
Manager
Joined: 14 Aug 2012
Posts: 56
Own Kudos [?]: 16 [0]
Given Kudos: 221
Location: United States
GMAT 1: 620 Q43 V33
GMAT 2: 690 Q47 V38
Send PM
Re: V04-39 [#permalink]
Wrong, it is the result of a study.

Can someone please clarify why it is a premise and not a conclusion?
Intern
Intern
Joined: 17 May 2018
Posts: 4
Own Kudos [?]: 0 [0]
Given Kudos: 12
Send PM
Re: V04-39 [#permalink]
I do not agree that the first boldface is a premise and not the conclusion.
Director
Director
Joined: 09 Mar 2018
Posts: 783
Own Kudos [?]: 453 [2]
Given Kudos: 123
Location: India
Send PM
Re: V04-39 [#permalink]
1
Kudos
1
Bookmarks
TariqOmar wrote:
I do not agree that the first boldface is a premise and not the conclusion.


Let me share my thoughts on this.( i was able to eliminate C)

(a)
Because A recent study indicates that people who keep their workout records are far more successful at achieving their fitness goals than people who don’t keep track of their workout. Therefore, Experts believe that many fail to achieve their fitness goals because they burn fewer calories than they intend to burn.

But if you rotate this

(b)
Because Experts believe that many fail to achieve their fitness goals because they burn fewer calories than they intend to burn. Therefore A recent study indicates that people who keep their workout records are far more successful at achieving their fitness goals than people who don’t keep track of their workout.

the second part of (b) could have used to keep a point, not the other way around

Generally a fact follows the opinion, Done in (a).

Let me know if this helps in some way.
Intern
Intern
Joined: 24 Aug 2017
Posts: 17
Own Kudos [?]: 8 [1]
Given Kudos: 212
Location: Canada
GPA: 3.67
Send PM
Re: V04-39 [#permalink]
1
Kudos
Here is why I think confusion arises, and why I think E is correct:

The first bold stmt of c) is true because it is a conclusion. while the OA from Bunel says it is "the conclusion". Clearly, saying that it is a conclusion, and saying that this is the conclusion of the prompt are two different very statements --> OA should be adjusted

What makes c) wrong is that the 2nd bold stmt doesn't back the results of the studies in the 1st bold stmt, it backs the opinion of the experts, which is the inference that the scientists have made from the conclusions of the study mentioned in the 1st leg (conclusions that serve as a premise)

Therefore, E) is correct.
Intern
Intern
Joined: 06 Aug 2017
Posts: 2
Own Kudos [?]: 6 [1]
Given Kudos: 137
Location: United States
Schools: Booth PT'24
Send PM
Re: V04-39 [#permalink]
1
Kudos
I agree with explanation. This is a Bold Face Question.
Intern
Intern
Joined: 18 Dec 2019
Posts: 28
Own Kudos [?]: 8 [0]
Given Kudos: 16
Send PM
Re: V04-39 [#permalink]
I guess the usage of the word "premise" in E is not the best usage. This is because it is what the recent study indicated and not a given.
GMAT Club Bot
Re: V04-39 [#permalink]
Moderator:
Math Expert
92922 posts

Powered by phpBB © phpBB Group | Emoji artwork provided by EmojiOne